Proving Int. of Even Powers of Sin

  • Thread starter Thread starter stripes
  • Start date Start date
  • Tags Tags
    even Sin
Click For Summary
The discussion focuses on proving the integral of even powers of sine, specifically the formula for the integral from 0 to π/2 of sin raised to the power of 2n. The user attempts to use integration by parts but finds the process increasingly complex and cumbersome. Suggestions include considering proof by induction to simplify the approach and acknowledging the need to manage the definite integral properly. The user expresses frustration with the limitations of their current coursework regarding integration techniques. The conversation highlights the challenges of tackling advanced calculus problems without sufficient foundational knowledge.
stripes
Messages
262
Reaction score
0

Homework Statement


should be my last question for at least the next few days...here goes...

Prove that, for even powers of sine,

\int^{\frac{\pi}{2}}_{0}sin^{2n}x dx = \frac{2\cdot4\cdot6\cdot...\cdot(2n - 1)}{2\cdot4\cdot6\cdot...\cdot2n}\cdot\frac{\pi}{2}

Homework Equations



<br /> uv - \int v du = \int u dvdx<br /> <br />

The Attempt at a Solution



let u = sin^{2n-1}x and dv = sin x dx
so du = (2n-1)(sin^{2n-2}x)(cos x)dx and v = -cos x

and we get:

\int sin^{2n}x dx = (-cos x)(sin^{2n-1}x) + (2n-1)\int (cos^{2}x)(sin^{2n-1}x)

and i used integration by parts again

let u = sin^{2n-2}x and dv = cos^{2}x dx
so du = (2n-2)(sin^{2n-3}x)(cos x)dx and v = \frac{1}{2}((sin x)(cos x) + x)

then we get:

\int sin^{2n}x dx = (-cos x)(sin^{2n-1}x) + \frac{2n-1}{2}(sin^{2n-2}(sinx cosx + x)) - (2n-2)\int ((sinx cosx + x)(sin^{2n-3}x)(cos x))dx

now I've realized I'm just pointlessly integrating by parts over and over...it's just getting harder and harder (and more difficult to put here on PF!)

If someone could guide me in the right direction for proving this formula, I would appreciate it. Thank you so much in advance!
 
Last edited:
Physics news on Phys.org


Did you forget that you have a definite integral?
 


In addition to remembering that this is a definite integral, so that you don't have that long sum, I would try proof by induction so that I only have to do the integration by parts once.
 


i knew all along that we have a definite integral, but we still need to get rid of that integral sign on the end of the whole thing, there'll always be that last integral...

our course hasn't taught us enough to integrate that...
 
Question: A clock's minute hand has length 4 and its hour hand has length 3. What is the distance between the tips at the moment when it is increasing most rapidly?(Putnam Exam Question) Answer: Making assumption that both the hands moves at constant angular velocities, the answer is ## \sqrt{7} .## But don't you think this assumption is somewhat doubtful and wrong?

Similar threads

Replies
6
Views
2K
  • · Replies 3 ·
Replies
3
Views
2K
  • · Replies 5 ·
Replies
5
Views
2K
Replies
3
Views
2K
  • · Replies 2 ·
Replies
2
Views
1K
  • · Replies 15 ·
Replies
15
Views
2K
  • · Replies 14 ·
Replies
14
Views
2K
  • · Replies 6 ·
Replies
6
Views
2K
Replies
3
Views
2K
Replies
12
Views
6K